LSAT and Law School Admissions Forum

Get expert LSAT preparation and law school admissions advice from PowerScore Test Preparation.

 Haleyeastham
  • Posts: 33
  • Joined: Aug 03, 2015
|
#19298
Having a very difficult time figuring out how D strengthens the argument- seems like it could even weaken it. Can you please clarify?

Thank you!
User avatar
 Dave Killoran
PowerScore Staff
  • PowerScore Staff
  • Posts: 5850
  • Joined: Mar 25, 2011
|
#19323
Hi Haley,

Thanks for the question! Let's first break down the argument, and then we'll take a closer look at answer choice (D).

The stimulus sets up a scenario where a local government ran an antismoking advertising campaign. This campaign was financed by taxing cigarettes at 20 cents per pack. A year later smoking was down 3%, and so the author concludes that the cause of the decline was the advertising campaign.

Now, the first thing to realize is that this is a causal argument. The author sees two things happening in succession, and concludes that the first caused the second. If we wanted, we could capture that using a causal diagram:

  • Cause ..... ..... ..... ..... ..... Effect

    antismoking advertising campaign :arrow: decline in number of smokers.
The relationship doesn't have to be diagrammed, though. I just did that so it was as clear as possible.

Second, when you read that argument, what's your reaction? Are you fully convinced that the author's conclusion is valid? Did you see any weakness in that argument? When I read that problem, I thought to myself, "How does the author know it's the advertising campaign that worked? If they raised the price by 20 cents, maybe the higher cost caused people to smoke less."

When I see that's it's a strengthen question, I'm not in trouble: I can still use the prephrased weakness that I saw. But instead of just using it as-is (which would hurt the argument), instead I would love an answer that eliminates that weakness. In basic causal arguments, if you eliminate an alternate cause, that strengthens the argument.

Answer choice (D) does just that. When I read the stimulus I saw an alternate cause for the conclusion, namely that it might have been the higher price that reduced the number of smokers. Answer choice (D) eliminates the higher price as a possible cause, because if the merchants absorbed the cost of the tax (which is essentially what happens when they reduce the price 20 cents), then the cost of a pack of cigarettes remained the same as before. If the price was the same, that means that cost wasn't a factor here. In abstract terms, (D) eliminates an alternate cause for the conclusion, and thus strengthens the argument.

Overall, this is a great question because it allows you to see how one of the fundamental reasoning/question type relationships on the LSAT works (strengthening a causal conclusion by eliminating an alt cause). So, I really want to make sure this makes total sense for you. If you have any questions, please let me know. Thanks!
 Johnclem
  • Posts: 122
  • Joined: Dec 31, 2015
|
#28008
Hi ,
Could someone please help me get rid of AC (B) for this one ? I got rid of it because I thought it may weaken but I wasn't 100% sure. I thought the word "substantial" weakens , since The conclusion used " a number of people". Also I didn't see how quitting smoking after the ad had ended would strengthen the conclusion.

Thanks
John
 Adam Tyson
PowerScore Staff
  • PowerScore Staff
  • Posts: 5153
  • Joined: Apr 14, 2011
|
#28079
Thanks for the question, John.

Answer B doesn't do anything to strengthen the argument because all it does it show additional effects - people reducing smoking in addition to those that quit. More effects have no impact on a causal argument, either positive or negative. To strengthen a causal argument you have to 1) eliminate an alternate cause, 2) show that when the cause is present, the effect is present, 3) show that when the cause is absent, the effect is absent, 4) suggest that the causal relationship might be reversed, or 5) do something to undermine the data, if the argument was based on data.

Stick to those five things and you can't go wrong! Since answer B provides none of those, it doesn't help. The cause could still be the money, or some other cause, and still some people could have reduced their smoking.

As to your question about quitting after the ad had ended, I'm not sure where you are seeing that idea in the argument or the correct answer choice, which is D (eliminating the money as a potential alternate cause). There's no suggestion, either in the stimulus or in that answer, that anything happened after the ad ended. In fact, I see no indication that the ad campaign ever ended - it could be ongoing. Still, even if the ad only ran for a week a year ago, it could still be that it caused a bunch of people to quit, and today they remain non-smokers.

I hope that clears things up for you!
 rpark8214
  • Posts: 23
  • Joined: Apr 27, 2017
|
#46146
Hi,

Is answer choice (A) incorrect because you cannot assume that the residents of the locality traded off other tobacco products with cigarettes? I had a difficult time with (A) because I thought it eliminated a potential alternate cause, and supported the fact that the decline was indeed due to the advertisements, not that residents were choosing other products like snuff/chewing tobacco instead of smoking.

Thank you!
User avatar
 Jonathan Evans
PowerScore Staff
  • PowerScore Staff
  • Posts: 726
  • Joined: Jun 09, 2016
|
#46763
Hi, Rpark,

Good question. Answer choice (A) obfuscates the issue by bringing up alternative tobacco products. You are correct that you could understand answer choice (A) as eliminating an alternate cause, that is, the decline in smoking is not because of people switching to other products; therefore it is more likely the ad campaign was a cause.

However, there are some problems with this answer choice. If people had switched to other tobacco products, this switch could have been the result of the ad campaign. After all, people would still technically be smoking less (dipping more?). Thus, eliminating this possibility does not necessarily directly strengthen the argument. You have to bring in additional assumptions to make answer choice (A) work.

In addition, the most glaring issue in the stimulus is whether the tax was the alternate cause. This is the key question that likely will be addressed in the credited response. Since answer choice (D) does a solid job eliminating the tax as the alternate cause, (D) is the best answer, the answer that strengthens this argument the most.

Get the most out of your LSAT Prep Plus subscription.

Analyze and track your performance with our Testing and Analytics Package.